Các bài toán về phương trình nghiệm nguyên bậc cao

11 1.9K 35
Các bài toán về phương trình nghiệm nguyên bậc cao

Đang tải... (xem toàn văn)

Tài liệu hạn chế xem trước, để xem đầy đủ mời bạn chọn Tải xuống

Thông tin tài liệu

Các bài toán về phương trình nghiệm nguyên bậc cao , các bài toán mũ bậc cao và cách ứng dụng bổ đề Hensel để giải nó . Các bạn và mọi người ở đây có thể thấy số học là một nghành quan trọng trong toán học , cùng với sự phát triển của các nghành giải tích , đại số , topo , giải tích phức thì số học vẫn giữ nguyên vai trò của nó . Rất nhiều nghành khác đã được sinh ra để phục vụ cho số học , ngày nay ta gọi là lý thuyết số . Một trong những dạng toán mà học sinh rất khó để xử lý là các phương trình nghiệm nguyên ở dạng mũ , bậc cao khiến người giải khó có cách để hạ bậc hoặc đưa về các phương trình đơn giản hơn ( định lý lớn Fermat là một ví dụ ) , bởi vậy tôi viết bài viết này để phục vụ cho mục đích đó . Dạng toán này khá khó , và mong bạn đọc đừng bỏ xót chi tiết nào , cũng xin nhắc thêm sẽ có một số bài toán chỉ nêu ý tưởng cho người đọc có thể tự giải chứ không phải tất cả các bài toán đều có lời giải sẵn ( nếu ở đây không nói là số thực ta hiểu tất cả các số ở đây là số nguyên ) , các phương pháp ở đây cũng chỉ là kinh nghiệm cá nhân tôi muốn chia sẻ , nó sẽ không phải là hay nhất , hy vọng bạn đã đọc hãy cố gắng tìm tòi nhiều lời giải mới đẹp đẽ và ngắn gọn hơn , làm đẹp thêm bài toán .Đặc biệt với 1 số trường hợp cần xét theo module nào đó để thấy tính chia hết của các số mũ , tôi sẽ chỉ ghi cần xét module nào và phần còn lại sẽ giành cho người đọc .Các bài toán sẽ đề tên địa chỉ lấy ( các web) và tác giả bài toán đó đăng lên hoặc người cung cấp nó cho tài liệu này . Cách ký hiệu , theo thứ tự A , B , C trong đó A là địa chỉ lấy bài toán , B là người đăng nó lên và C là nó được trích từ đề thi nào ( trong phạm vi hiểu biết ) Để có thể giải được các bài toán này ( không nhất thiết là phương trình nghiệm nguyên ) ta cần biết một bổ đề quan trọng sau , được gọi là bổ đề Hensel hoặc nói cách khác là định lý LTE , nó được phát biểu như sau : Cho p là một số nguyên tố , ký hiệu v_p_(x) là số mũ cao nhất trong phân tích của x đối với p Khi đó nếu với hai số nguyên khác nhau a,b , số nguyên dương n và thỏa mãn v_p_(a-b) > 0 thì ta luôn có đẳng thức , hầu hết ta chỉ xét cho p lẻ Là v_p_(a-b) + v_p_(n)=v_p_(a^n-b^n) Trong trường hợp n là số lẻ , ta có thể thay a-b bởi a+b , và lưu ý là cả a,b đều không chia hết cho p Trong trường hợp p = 2 và n là số nguyên dương chẵn , và 2 | x –y ta có V_2_(x^n-y^n)=v_2_(x+y)+v_2_(x-y)+v_2_(n)-2 Chứng minh không nêu ở đây , bài viết chỉ mang tính áp dụng , nếu không bạn có thể xem ở đây : http://diendantoanhoc.net/forum/index.php?/topic/67575-%E1%BB %A9ng-d%E1%BB%A5ng-s%E1%BB%91-mu-l%E1%BB%9Bn-nh %E1%BA%A5t-c%E1%BB%A7a-th%E1%BB%ABa-s%E1%BB%91- nguyen-t%E1%BB%91-trong-cac-bai-toan-s%E1%BB%91-h%E1%BB %8Dc/ * Bài toán 1 ( VMF , bangbang1412 ) Cho x,y là các số nguyên dương , p là một số nguyên tố , tìm tất cả x,y,p thỏa mãn đẳng thức X^3 + y^3 =p^4 Trước hết ta xét cho p là số nguyên tố lẻ .Ta có thể thấy do p là số nguyên tố , nên bắt buộc phải có p|x+y , vì lũy thừa của x,y là lẻ nên ta có thể áp dụng định lý LTE ( lưu ý rằng ở đây ta chỉ xét cho gcd(x,p)=gcd(y,p)=1 ) Ta có ngay v_p_(3)+v_p_(x+y)=v_p_(x^3+y^3)=4 Có thể thấy với p=3 phương trình không có nghiệm nguyên dương do đó v_p_(3)=0 , vì vậy ta có v_p_(x+y)=v_p_(x^3+y^3)=4 vì vậy phải có x+y>= x^3+y^3 , điều này là vô lý . Vậy bắt buộc ta phải có p chẵn , tức là p=2 . Khi đó kiểm tra thấy có duy nhất bộ (x,y,p)=(2,2,2) là thỏa mãn bài toán . * Bài toán 2 ( Trần Quốc Luật ) Giải phương trình nghiệm tự nhiên 33^x + 31 = 2^y Trước hết ta có thể thấy y>=5 do 33^x+31>=32=2^5 Nếu y=5 ta có ngay x=0 Đưa phương trình về dạng 33^x – 1 = 2^y – 2^5 Để ý rằng do y>=5 nên v_2_(2^y-2^5)=5 V_2_(33^x-1)=v_2_(2^y-2^5)=5 Hay 5+v_2_(x)=5 do đó v_2_(x)=0 nên x là số lẻ Đặt x=2k+1 để ý rằng 33^2 chia cho 64 dư 1 nên ta có phương trình 33^(2k+1)+31=2^y Ta lại nhận thấy rằng số 33^(2k+1)=(32+1)^(2k+1) * Bài toán 3 : Tìm các số m,n,k nguyên dương thỏa mãn đẳng thức 5^m + 7^n = k^3 Bài toán này khá khó , tuy nhiên ý tưởng nó cũng khá rõ ràng , vì ta sẽ đưa nó về dạng chuẩn để áp dụng được bổ đề Hensel Bằng cách xét module khá cơ bản , ở đây ta chọn module của 8 , sẽ thu được ngay m chẵn và n lẻ Tiếp theo ta nhận thấy k^3 chỉ chia 7 dư 0,1,6 trong khi đó 5^m + 7^n sẽ chia 7 dư 1 nếu 3|m , sẽ chia 7 dư 2 nếu m=3a+1 và chia 7 dư 4 nếu m=3a+2 Do đó ta phải có 3|m , đặt m=3a và ta thu được phương trình 7^n = k^3 – 5^(3a) Nhận thấy nếu k – 5^a không chia hết cho 7 thì nó phải là 1 vì vế trái là lũy thừa của 7 nguyên tố . Nếu k = 5^a + 1 ta có k^3-5^(3a)=1+3.5^a.(5^a+1)=7^n nên ta có 7^n – 1 = 3.5^a.(5^a+1) Đặt 5^a = x ta có 7^n – 1 = 3x(x+1) Vì n lẻ nên đặt n=2b+1 ta sẽ xét chẵn lẻ theo b . Ta có 7^n = 7^(2b+1) = 49^b . 7 = (50-1)^b.7 Nếu b lẻ ta có 7^n \equiv -7 (mod5) và do đó 7^n -1 \equiv -8 (mod5) và do đó ta có điều vô lý vì vế phải chia hết cho 5 Nếu b chẵn ta có VT \ equiv 6 (mod5) cũng vô lý ( các sự vô lý ở đây do có a > 0 ) Vì vjaay ta phải có k – 5^a chia hết cho 7 Áp dụng định lý LTE ta có : V_7_(k-5^a)+v_7_(3)=v_7_(k-5^a)=v_7_(k^3- 5^(3a))=n > 0 Do đó ta có k – 5^a = k^3 – 5^(3a) hoàn toàn vô lý Vậy phương trình không có nghiệm nguyên dương Trong trường hợp yêu cầu nghiệm tự nhiên ta sẽ có nghiệm (m,n,k)=(0,1,2) * Bài toán 4 ( VMF , Juliel ) Giả sử số nguyên tố p lẻ có phân tích p = a+b với a,b là các số nguyên dương và tồn tại số nguyên dương n thỏa mãn n | a^n + b^n . Chứng minh rằng n chia hết cho p Dễ dàng thấy do p nguyên tố nên phải có gcd(a,p)=gcd(a,b)=gcd(b,p)=1 Do p lẻ nên nên a+b lẻ và a^n +b^n lẻ , do đó n lẻ Gọi u là ước nguyên tố nhỏ nhất của n , ta có a^n + b^n \equiv 0(modu) Gọi t là số mà thỏa mãn at \equiv 1(mod u) ( dễ thấy gcd(a,u)=1 nên ta luôn có sự tồn tại của t ) Ta có (at)^n + (bt)^n \equiv 0(modu ) nên (bt)^n \equiv -1(modu) Lại có n lẻ nên (-bt)^n \ equiv 1(mod u ) Gọi x là cấp của –bt theo module u ta có (-bt)^x \equiv 1(modu) Theo tính chất của cấp ta phải có x|n Lại theo định lý nhỏ fermat ta có (-bt)^(u-1) \equiv 1(mod u) nên x|u-1 Nếu x>1 thì x có một ước nguyên tố nào đó là k , mà x|u-1 nên k < u-1 nên n có một ước nguyên tố k < u , trái với điều giả sử Do đó ta có x=1 và vì vậy –bt \equiv 1(modu) Cộng với at\equiv 1(modu) ta có ngay p.t\equiv 0(modu) do gcd(t,u)=1 nên u=p và ta có ngay n chia hết cho p * Bài toán 5 ( VMF , LNH ) Tìm nghiệm nguyên dương của phương trình 3^m - 7^n =2 Đây là một bài toán khá khó , tôi đã mất khá lâu thời gian có thể giải được nó tuy nhiên có một điều là nó không thể đưa về định lý LTE để giải , mà ở đây tôi chỉ xét module đơn giản cùng với nhị thức newton và phương pháp quy nạp toán học . Trước hết Nếu m chẵn , đặt m=2k ta có 9^k – 2 = 7^n Ta có 9^k – 2 \equiv -1 \equiv 7 (mod8) nên 7^(n-1) \equiv 1(mod8) Nếu n lẻ ta có n=2a+1 và ta có 7^(n-1)=7^(2a) \equiv 1(mod8) còn n chẵn thì hiển nhiên sẽ không thỏa mãn Với cách đặt như thế ta sẽ thu được phương trình 9^k - 2 = 7^(2a+1) Do ở đây chỉ là nghiệm nguyên dương , xét k=1 dễ dàng cho nghiệm n=1 Nếu k>=2 ta có 9^k chia hết cho 81 . Do vậy 7^(2a+1) +2 phải chia hết cho 81 Ta sẽ sử dụng quy nạp để chứng minh rằng 7^(2a+1) + 2=7.49^a+2 không chia hết cho 81 với mọi a nguyên dương . Với a =1 ta có khẳng định đúng Giả sử khẳng định đúng đến a=x ta sẽ chứng minh nó đúng với a=x+1 Ta xét 7.49^(x+1)+2 nếu nó chia hết cho 81 thì trước tiên phải chia hết cho 9 đã , ta có 7.49^(x+1)+2\equiv -2.4^(x+1)+2=2(1-4^(x+1))(mod9) Do đó ta có 9|4^(x+1)-1 Ta sẽ kiểm tra các điều kiện để một số có dạng 4^y -1 chia hết cho 9 Nếu y là bội của 3 ta có sự thỏa mãn Nếu y = 3u+1 ta có 4^y-1=4.(63+1)^y-1\equiv 3(mod9) Tương tự với y=3u+2 và do đó nếu quy nạp sai đến x+1 thì phải có x+1 chia hết cho 3 . Đặt x+1=3b Ta có 7.49^(3b)+2=7^(6b+1)+2 = ( 6+1)^(6b+1)+2 Sử dụng khai triển newton ta thấy nó chỉ chia hết tối đa cho 3 mà thôi , do đó ta có điều vô lý và vì vậy trong TH này ta có (k,n)=(1,1) Nếu m là số lẻ Sử dụng xét module 8 ta chứng minh được n chẵn ( bằng cách làm hoàn toàn tương tự người đọc có thể giải quyết được TH này , lưu ý 49^3 có dạng 9u+1) Bài toán được giải quyết • Bài toán 6 ( VMF , Juliel ) Tìm nghiệm nguyên của phương trình Y^(2010) = x^(2010) – x^(1340)-x^(670)+2 Để ý rằng các số 2010 , 1340 và 670 đều là bội của 670 nên có thể đặt 670=a và thu được y^(3a)=x^(3a)-x^(2a)-x^a + 2 Tiếp tục đặt y^a = u , x^a = v ta có u^3 = v^3-v^2-v+2 Bài toán này từ đây được giải quyết khá dễ bởi nguyên lý kẹp • Bài toán 7 ( VMF , Juliel ) Trước hết ta định nghĩa một số nguyên dương được gọi là số square-free nếu nó là tích các số nguyên tố phân biệt có lũy thừa bậc 1 trong nó Tìm tất cả các số nguyên dương a sao cho a^(a-1) – 1 là một số square- free Trước hết giả sử số a^(a-1)-1 là một số square – free khi đó nếu a=1 ta thấy không thỏa mãn , a=2 không thỏa mãn nên a>2 , do đó a-1>1 Vì vậy gọi p là một ước nguyên tố nào đó mà p|a-1 Áp dụng định lý LTE ta có V_p_(a-1)+v_p_(a-1)=v_p_(a^(a-1)-1)=1( theo định nghĩa số square –free) Mặt khác 2v_p_(a-1) >=2 nên ta có điều vô lý Vậy với mọi a thì số a^(a-1) – 1 không là số square – free • Bài toán 8 : ( PSW của diendantoanhoc ) Giải phương trình nghiệm nguyên dương (x!)^k + (y!)^k = (k+1)^n . (n!)^k Ngoài cách sử dụng định lý LTE ( xem chứng minh ở http://diendantoanhoc.net/forum/index.php?/topic/2531-gi%E1%BA%A3i- pt-nguy%C3%AAn-d%C6%B0%C6%A1ng-xkykk1n-cdot-nk/ ) Có thể chia 2 vế cho (n!)^k và biện luận khi đó khá dễ dàng không cần phức tạp như lời giải của đường link nói trên  Với một bài toán khó như vậy , hy vọng bạn sẽ không bỏ qua nó , ta cùng sang các bài toán trong một số đề thi , trước hết tôi đề xuất phương trình với các cách giải không phụ thuộc nhiều định lý LTE , và biến hóa hơn . • Bài toán 9 ( Trần Quốc Luật ) Tìm các số nguyên dương x,y thỏa mãn phương trình 2^(3^x) + + 1 = 19.3^y Số mũ ở vế trái là lẻ , ta nhận thấy rằng nó đã đủ điều kiện sử dụng định lý LTE Áp dụng ta có v_3_(2^(3^x)+1)=v_3_(3^x)+v_3_(2+1)=x+1 Bởi vậy mà y=x+1 Từ đó thu được phương trình 2^(3^x)+1=19.3^(x+1) Hay 2^(3^x)+1=57.3^x Đặt 3^x=t ta có 2^t + 1 = 57t Bằng quy nạp ta chứng minh được nếu t >= 10 thì phương trình trên không thỏa mãn do 2^t + 1 > 57t Vậy ta có t <= 9 tức là x <=2 , do vậy có x=1 và x=2 Do đó thu được duy nhất bộ số (x,y)=(2,3) thỏa mãn bài toán • Bài toán 10 : ( VMF , HoangTung126 , VMO 2004 ) Tìm tất cả x,y,z nguyên dương thỏa mãn phương trình (x+y)(xy+1)=2^z Không mất tính tổng quát giả sử x <= y Do vế phải là lũy thừa của một số nguyên tố nên các phần tử của vế trái lần lượt là lũy thừa của nó . Do vậy đặt x+y = 2^m và xy+1=2^(z-m) với m>0 , dễ thấy z>=2m do xy+1>=x+y với mọi x,y nguyên dương . Nếu x=1 ta có y+1=2^m và y+1=2^(z-m) bởi vậy có z=2m Nghiệm đầu tiên sẽ là (x,y)=(1,2^m – 1) với z=2m , z,m nguyên dương tùy ý Đến đây ta hạn chế chỉ xét cho x,y>=3 ( TH kia x,y không thể chẵn nên loại ) Ta có x^2 -1 = x(x+y)-(xy+1) chia hết cho 2^m ( do z>=2m) Vì x,y gcd(x-1,x+1)=2 do x lẻ nên để dễ hình dung ta sẽ đặt x-1=2^a.u x+1=2^b.v Trong đó các số này nguyên dương thỏa mãn a+b>=m và u,v lẻ Khi đó hiển nhiên nếu a,b>=2 thì gcd(x-1,x+1)>=4 vì vậy nhất định một trong hai số a,b là 1 , khi đó số còn lại sẽ không nhỏ hơn m-1 Do đó ta có hoặc là 2^m | x-1 hoặc 2^m | x+1 Theo giả sử x<=y, ta có thể xét x=y thu được 2x(x^2+1)=2^z vô lý do x(x^2+1) có ước nguyên tố lẻ Do đó x<y Ta có 0 < x-1 < [ (x+y)/2] -1 < 2^(m-1) do x+y=2^m Do đó ta có 2^(m-1)|x+1 Do x,y cùng lẻ nên ta có y khác x+1 nên y>=x+2 , ta lại có x+y <= [ (x+y-2)/2 ] +1 =2^(m-1) , và hiển nhiên ta có ngay x+1=2^(m-1) Từ đó thu được các bộ nghiệm (x,y,z)=(1,2^m – 1 , 2m) , ( 2^m – 1 , 1 , 2m) , (2^(m-1) -1 , 2^(m-1)+1,3m-2) , ( 2^(m-1)+1,2^(m-1)-1,3m-2) với m>=3 nguyên dương . • Bài toán 11 : ( VMF , Jinbe ) , giải phương trình nghiệm nguyên dương 7^x + 1= 5^y + 3^z Sử dụng cách xét theo module 8 . Trước tiên nếu x chẵn , chúng ta sẽ chứng minh được rằng y,z cũng chẵn Ta sẽ thấy cả x,y,z đều chẵn trong trường hợp này Đổi sang bộ số (x,y,z)=(2a,2b,2c) Khi đó có phương trình 49^a + 1 = 25^b + 9^c Tuy nhiên 49^a +1 \equiv 2(mod3) còn 25^b+9^c \equiv 1(mod3) nên phương trình trong trường hợp này chỉ có nghiệm (0,0,0) Vậy x lẻ , sử dụng cách xét như trên ta thu được x,y,z đều lẻ Đổi sang bộ số (x,y,z)=(2a+1,2b+1,2c+1) Dễ dàng giải quyết nó với trường hợp một trong các số a,b,c là 0 Vậy ta xét a,b,c>0 khi đó có phương trình 7.49^a + 1 = 5.25^b + 3.9^c Ta có 7.49^a + 1\equiv (-1)^a. 7 + 1 (mod 5) Và 5.25^b + 3.9^c \equiv 3.(-1)^c (mod 5) Bởi vậy cân có 7. (-1)^a + 1 \equiv 3.(-1)^c ( mod 5) Chỉ xảy ra khi a,c cùng chẵn . • Bài toán 12 : ( Romanian IMO TST 2005 ) Giải phương trình nghiệm nguyên dương : 3^x = 2^x . y + 1 Nếu x lẻ , ta có 3^x – 1 = 2^x .y Áp dụng định lý LTE ta có X= V_2_(3^x-1)=v_2_(x)+v_2_(3-1)=1 Nên ta có ngay x=1 , từ phương trình ta có ngay y=1 Nếu x chẵn , ta sẽ đưa ra 1 dạng khác của định lý LTE , với p chẵn như sau Nếu n là một số nguyên dương chẵn , và x,y nguyên dương thỏa mãn 2|x- y hoặc 2|x-y Khi đó ta luôn có đẳng thức V_2_(x^n – y^n ) = v_2_(x-y) + v_2_(x+y)+v_2_(n)-1 Áp dụng với (x,y,n)=(3,1,x) ta có V_(2^x.y)=V_2_(3^x-1)=v_2_(3-1)+v_2_(3+1)+v_2_(x)- 1=2+v_2_(x) Đặt x=2^m . k với k lẻ , m,k nguyên dương ta sẽ chứng minh quy nạp 2^m.k >= m+2 với m >=3 Từ đó thu được nghiệm của phương trình (x,y)=(1,1),(2,2),(4,5) • Bài toán 13 : ( IMO shorlist 2010 ) Tìm tất cả m,n không âm thỏa mãn đẳng thức M^2 + 2.3^n = m.(2^(n+1)-1) Lập các giá trị của n từ 0 đến 5 Ta có: - Nếu n = 0 không có m thỏa mãn . - Nếu n = 1 không có m thỏa mãn . - Nếu n = 2 không có m thỏa mãn . - Nếu n = 3 thì m = 6 hoặc m = 9 - Nếu n = 4 thì không có m thỏa mãn . - Nếu n = 5 thì m = 9 hoặc m = 54 . Giờ ta sẽ giải quyết với n > = 6 Nhận thấy phải có m | 2.3^n nên m có dạng 2.3^p hoặc 3^p với 0 <= p <= n Đặt q = n – p ta có ( m = 3^p ) 2^(n+1) – 1 = 3^p + 2.3^q Bởi vậy ta có 3^p < 2^(n+1) = 8^ [ (n+1)/3 ] < 9^[ (n+1)/3 ] = 3^[2(n+1)/3] Và 2.3^q < 2^(n+1) = 2. 8^[n/3] < 2 . 9^[n/3] < 2.3^[2(n+1)/3] Do vậy ta có p,q < 2(n+1)/3 Ta có n - p < 2(n+1)/3 nên 3n – 3p < 2n+2 hay (n-2)/3 < p Tương tự thì (n-2)/3 < q Do vậy ta có (n-2)/3 < p,q < 2(n+1)/3 Vì n > 6 nên min (p,q) > 1 do đó p,q>=2 nên 2^(n+1) – 1 là một bội của 9 Kiểm tra và thấy rằng ord_9_(2)=6 nên 6 | n+1 Đặt n + 1 = 6a ta có 2^(n+1) – 1 = 4^(3a)-1 = (2^a-1)(2^a+1)(4^(2a) +4^a+1) là bội của 9 Nhưng 4^(2a)+4^a+1=(4^a-1)^2+3.4^a có (4^a-1)^2 chia hết cho 9 còn 3.4^a chỉ chia hết cho 3 nên v_3_(4^(2a)+4^a+1)=1 Từ phương trình 2^(n+1) – 1 = 3^p + 2.3^q Ta thấy 2^(n+1) – 1 phải chia hết cho 3^[min(p,q)] mặt khác có v_3_(4^(2a)+4^a+1)=1 nên phải có (2^a-1)(2^a+1) chia hết cho 3^[min(p,q)-1] Chỉ có 1 trong 2 số 2^a-1 và 2^a+1 là chia hết cho 3 nên hoặc 3^[min(p,q)- 1] | 2^a + 1 hoặc là 3^[min(p,q)-1] | 2^a – 1 Nên ta có 3^[ min (p,q)-1] <= 2^a + 1 <= 3^a = 3^[ (n+1)/6] nên ta có (n-2)/3 <Min(p,q) < 1+(n+1)/6 Bởi vậy ta có 2(n-2) < 6 + (n+1) hay n < 11 , đến đây dễ dàng giải quyết tiếp Với n có dạng 2.3^p ta xử lý tương tự và chứng minh được phương trình có các nghiệm (m,n) = (6,3) , (9,3) , (9,5) , (54,5) . • Bài toán 14 : Tìm tất cả các số nguyên dương x,y thỏa mãn 2^x + 1 = x^2.y Đặt x = 3^k . d với gcd(3,d)=1 Hiển nhiên x lẻ nên d lẻ , sử dụng định lý LTE ta có : V_3_(2^x+1)=v_3_(x^2.y) Hay v_3_(x)+v_3_(2+1)=v_3_(3^(2k).d.y) Nên ta có k + 1 = 2k + v_3_(y) => 1 = k + v_3_(y) Giả sử d > 1 thì d có ước nguyên tố . Nên k = 0 hoặc k = 1 , ta có 2^x + 1 chia hết cho d chọn p là ước nguyên tố nhỏ nhất của d , khi đó ta có 2^x \equiv -1(mod p) 2^(2x) \equiv 1(modp) Theo định lý nhỏ Fermat ta có 2^(p-1)\equiv 1(modp) Gọi u là cấp của 2 theo module p (vì gcd(p,2)=1) 2^u \equiv 1(modp) Theo tính chất của cấp ta có u|p-1 và u|2x nên u|2.3^k.d Ta có u <= p – 1 < p là ước nguyên tố nhỏ nhất của d , mà u > 1 nên gcd(u,d)=1 do đó u|2.3^k nên u|6 . Nếu u=2 thì p|3 vô lý do gcd(d,3)=1 , nên u= 3 hoặc u = 6 do đó p| 7 hoặc p|63 , kiểm tra lại đều thấy không thỏa mãn . Nên d = 1 và k = 0 hoặc 1 từ đó tìm được nghiệm (x,y)=(1,3),(3,1) • Bài toán 15 ( AoPs , Altricona ) Tìm tất cả các số nguyên không âm thỏa mãn 9 + 5^a = 3^b + 7^c Sau đây là lời giải của tôi : Thử qua thử lại chúng ta thấy có các nghiệm (a,b,c)=(0,1,1),(2,3,1) ,(0,2,0) Cũng xin nhắc lại lưu ý các phương trình nghiệm nguyên bậc cao nếu là giải phương trình mà không phải đưa về các dạng phương trình phụ trợ ( phương trình Pell , phương trình Pitago , …… ) thì nó có rất ít nghiệm và thường là nghiệm nhỏ có thể kiểm soát được , các số xuất hiện trong bài ta thường xét module tích của một vài số , lấy module gần nhất là + 1 hoặc -1 của một số hạng của nó , trong các bài toán trên bạn sẽ thấy tôi luôn cố gắng xét các module như vậy . Đây là một bài toán khó , không thể dùng tính chẵn lẻ để giải quyết nó , nhưng tôi lại có một cách giải quyết rất hay như sau , như đã nói ở trên các module luôn là +1 hoặc -1 , để ý là 9=2.4+1,5=1.4+1,3=4-1,7=2.4-1 Ta thấy rằng 9 + 5^a \equiv 2(mod4) Và 3^b + 7^c \equiv -2 \equiv 2(mod4) khi b,c lẻ 3^b+7^c \equiv 0 (mod4) nếu có 1 số lẻ một số chẵn 3^b + 7^c \equiv 2 (mod4) nếu cả hai cùng chẵn . Vậy b,c cùng tính chẵn lẻ . Nhưng lại thấy rằng 3^b + 7^c \equiv 1(mod3) nên phải có a chẵn thì mới thu đươc 5^a \equiv 1(mod3) Sau đây ta chỉ quan tâm đến các nghiệm nguyên dương . Nếu a,b,c cùng chẵn và khác không ta chuyển bộ (a,b,c)->(2x,2y,2z) thu được phương trình . 9+ 25^x = 9^y + 49^z Ta có 25^x + 9 \equiv 4 (mod5) nên 9^y + 49^z\equiv 4 (mod5) Mặt khác 9^y+49^z\equiv (-1)^y + (-1)^z (mod5) Nhưng dù y,z chẵn lẻ như thế nào cũng không thu được (-1)^y+(- 1)^z\equiv 4( mod5) nên bắt buộc phải có một số là 0 . Nếu x = 0 thì rất dễ giải quyết , nếu y = 0 thì 8 + 25^x = 49^z có một số lẻ và một số chẵn Nếu z = 0 thì 8 + 25^x = 9^y cũng không xảy ra do hai vế không đồng tính chẵn lẻ Vậy trường hợp này được giải quyết , kết luận nghiệm của nó là (a,b,c)=(0,2,0) Nếu a chẵn và b,c lẻ chuyển bộ (a,b,c)->(2x,2y+1,2z+1) thu được phương trình : 9 + 25^x = 3.9^y + 7.49^z Trước hết nếu x= 0 ta có thể tự giải quyết còn trường hợp còn lại như đã nói chỉ xét cho nghiệm dương ( chú ý nghiệm (a,b,c)=(2,3,1) ) Ta có 3.9^y + 7.49^z \equiv (-1)^y.3 + (-1)^z.7 (mod5) [...]... cho người đọc ) Như vậy ta chỉ còn việc giải quyết phương trình 25^x = 18 + 7.49^(2n) Phương trình này đến đây đã khá cơ bản , giải quyết nó giành cho người đọc Kết luận nghiệm (a,b,c)=(0,2,0);(0,1,1);(2;3;1) Còn đây là bài toán cuối cùng tôi muốn các bạn tự giải , khá tổng quát Giải phương trình nghiệm nguyên dương , trong đó p nguyên tố , n là số nguyên thỏa mãn gcd(n,p)=1 A^n + b^n = p^k Với a,b,n,p,k . Các bài toán về phương trình nghiệm nguyên bậc cao , các bài toán mũ bậc cao và cách ứng dụng bổ đề Hensel để giải nó . Các bạn và mọi người ở đây có thể thấy. nghiệm (a,b,c)=(0,1,1),(2,3,1) ,(0,2,0) Cũng xin nhắc lại lưu ý các phương trình nghiệm nguyên bậc cao nếu là giải phương trình mà không phải đưa về các dạng phương trình phụ trợ ( phương trình. Một trong những dạng toán mà học sinh rất khó để xử lý là các phương trình nghiệm nguyên ở dạng mũ , bậc cao khiến người giải khó có cách để hạ bậc hoặc đưa về các phương trình đơn giản hơn (

Ngày đăng: 10/09/2014, 21:35

Từ khóa liên quan

Tài liệu cùng người dùng

Tài liệu liên quan